Categories
Economists Fields Harvard

Harvard. 13 Ph.D. Candidates, General or Special Examinations by Field, 1912-13

 

For thirteen Harvard economics Ph.D. candidates this posting provides information about their respective academic backgrounds, the six subjects of their general examinations along with the names of the examiners, the subject of their special subject, thesis subject and advisor(s) (where available). This transcribed announcement is for the academic year 1912-13.

________________________________________

DIVISION OF HISTORY AND POLITICAL SCIENCE
EXAMINATIONS FOR THE DEGREE OF PH.D.
1912-13

Notice of hour and place will be sent out three days in advance of each examination.
The hour will ordinarily be 4 p.m.

Charles Edward Persons.

Special Examination in Economics, Wednesday, January 15, 1913.
General Examination passed February 25, 1909.
Academic History: Cornell College (Iowa), 1898-1903; Harvard Graduate School, 1904-05, 1906-09. A.B., Cornell, 1903; A.M., Harvard, 1905. Instructor in Economics, Wellesley, 1908-09; Preceptor in Economics, Princeton, 1909-10; Instructor in Economics, Northwestern, 1910-12; Assistant Director, St. Louis School of Social Economy, Washington University, 1913-.
General Subjects: 1. Economic Theory and its History. 2. Economic History to 1750. 3. Economic History since 1750. 4. Sociology and Social Reform. 5. Transportation and Foreign Commerce. 6. History of American Institutions.
Special Subject: Transportation.
Committee: Professors Taussig (chairman), Bullock, Ripley and Rappard.
Thesis Subject: “The History of the Ten-Hour Law in Massachusetts.”
Committee on Thesis: Professors Taussig, Bullock, and Ripley.

Clyde Orval Ruggles.

Special Examination in Economics, Friday, January 17, 1913.
General Examination passed May 20, 1909.
Academic History: Hedrick Normal School, 1895-96; Iowa State Normal School and Teachers’ College of Iowa, 1901, 1903-06; State University of Iowa, 1906-07; Harvard Graduate School, 1907-09. A.B., Teachers’ College, 1906; A.M., State University, 1907. Professor of Economics, State Normal School, Winona, Minn., 1909-13.
General Subjects: 1. Economic Theory and its History. 2. Sociology and Social Reform. 3. Statistics. 4. Economic History to 1750, with especial reference to England. 5. Money, Banking, and Commercial Crises. 6. History of American Institutions.
Special Subject: Money and Banking.
Committee: Professors Bullock (chairman), Sprague, Turner, and Dr. Day.
Thesis Subject: “The Economic Basis of the Greenback Movement in Iowa and Wisconsin.”
Committee on Thesis: Professors Sprague, Turner, and Dr. Day.

Harold Hitchings Burbank.

General Examination in Economics, Monday, April 28, 1913.
Committee: Professors Bullock (chairman), Channing, Taussig, Gay, and Dr. Day.
Academic History: Dartmouth College, 1905-10; Harvard Graduate School, 1911-13. A.B., Dartmouth, 1909; A.M. ibid., 1910. Instructor in Economics, Dartmouth, 1910-11; Assistant in Economics, Harvard, 1911-12; Instructor in Economics, 1912-13.
General Subjects: 1. Economic Theory and its History. 2. Economic History since 1750. 3. Money, Banking, and Crises. 4. Public Finance and Financial History. 5. Tariff History and International Trade. 6. History of American Institutions.
Special Subject: Taxation.
Thesis Subject: “The History of the General Property Tax in Massachusetts since 1775.” (With Professor Bullock.)

John Alvin Bigham.

General Examination in Economics, Wednesday, April 30, 1913.
Committee: Professors Carver (chairman), Bullock, Cole, Fite, and Dr. Copeland.
Academic History: University of Kansas, 1904-08; Harvard Graduate School, 1908-10, 1911-12. A.B., Kansas, 1908; A.M., Harvard, 1909. Instructor in Economics, St. Augustine’s School, Raleigh, N.C., 1910-11.
General Subjects: 1. Economic Theory and its History. 2. Economic History since 1750. 3. Sociology and Social Reform. 4. Public Finance and Financial History. 5. Economics of Agriculture. 6. History of American Institutions.
Special Subject: Economics of Agriculture, with especial reference to American conditions.
Thesis Subject: (undecided).

John Ise.

General Examination in Economics, Friday, May 2, 1913.
Committee: Professors Bullock (chairman), Wyman, Carver, Sprague, and Dr. Copeland.
Academic History: University of Kansas, 1904-11; Harvard Graduate School, 1911-13. MUS.B, Kansas, 1908; A.B., ibid., 1910; LL.B., ibid., 1911; A.M., Harvard, 1912. Assistant in Economics, 1912-13.
General Subjects: 1. Economic Theory and its History. 2. Economic History since 1750. 3. Sociology and Social Reform. 4. Public Finance and Financial History. 5. Money, Banking, and Crises. 6. Jurisprudence.
Special Subject: Public Finance.
Thesis Subject: “The Government Land Policy since 1880.” (With Professor Bullock.)

Lloyd Morgan Crosgrave.

General Examination in Economics, Wednesday, May 7, 1913.
Committee: Professors Taussig (chairman), Ripley, Bullock, Fite, and Dr. Copeland.
Academic History: Indiana University, 1905-09; Harvard Graduate School, 1910-13. A.B., Indiana, 1909; A.M., Harvard, 1911. Teacher of History, Decatur High School, Ill., 1909-10; Instructor in Economics, Harvard, 1912-13.
General Subjects: 1. Economic Theory and its History. 2. Economic History since 1750. 3. Statistics. 4. Public Finance and Railroads. 5. Labor Problems, including Social Reforms. 6. History of American Institutions since 1789.
Special Subject: Labor Problems.
Thesis Subject: “The American Glass Industry.” (With Professor Taussig.)

Lucius Moody Bristol.

Special Examination in Economics (Social Ethics), Thursday, May 8, 1913.
General Examination passed May 4, 1911.
Academic History: University of North Carolina, 1894-95; Boston University School of Theology, 1896-99; Harvard Divinity School, 1909-10; Harvard Graduate School, 1910-11. A.B., North Carolina, 1895; S.T.B., Boston University, 1899; A.M., Harvard, 1910. Assistant in Economics, Harvard, 1911-13; Instructor in Sociology and Applied Christianity, Tufts, 1910-12; Assistant Professor of Applied Christianity, Tufts, 1912-13.
General Subjects: 1. Ethical Theory. 2. Economic Theory. 3. Labor Problems. 4. Social Reforms. 5. Sociology. 6. Statistics.
Special Subject: Sociology.
Committee: Professors Carver (chairman), Taussig, Bullock, and Dr. Brackett.
Thesis Subject: “The Development of the Doctrine of Adaptation as a Theory of Social Progress.” (With Professor Carver.)
Committee on Thesis: Professors Carver, Sprague, and Dr. Brackett.

Yamato Ichihashi.

Special Examination in Economics, Friday, May 12, 1913.
General Examination passed May 1, 1912.
Academic History: Leland Stanford Junior University, 1904-08; Harvard Graduate School, 1910-12; A.B., Leland Stanford, 1907; A.M., ibid., 1908. Assistant in Economics, Leland Stanford, 1908-10.
General Subjects: 1. Economic Theory and its History. 2. Economic History since 1750. 3. Sociology and Social Reform. 4. Statistics. 5. Anthropology. 6. Labor Problems and Industrial Organization.
Special Subject: Labor Problems.
Committee: Professors Ripley (chairman), Taussig, Carver, and Dr. Day.
Thesis Subject: “Emigration from Japan, and Japanese Immigration into the State of California.” (With Professor Ripley.)
Committee on Thesis: Professors Ripley, Turner, and Carver.

George Henry von Tungeln.

General Examination in Economics (Social Ethics), Wednesday, May 14, 1913.
Committee: Dr. Ford (chairman), Professors Taussig, Turner, R.B. Perry, Drs. Brackett and Foerster.
Academic History: Central Wesleyan College, 1904-06, 1907-09; Northwestern University, 1909-10; Harvard Graduate School, 1911-13. Ph.B., Central Wesleyan, 1909; A.M., Northwestern, 1910.
General Subjects: 1. Ethical Theory. 2. Economic Theory. 3. Poor Relief. 4. Social Reforms. 5. Sociology. 6. Criminology and Penology.
Special Subject: Criminology and Penology.
Thesis Subject: “Boston Juvenile Offenders in their Economic and Moral Relations.” (With Professor Peabody and Dr. Ford.)

Eliot Jones.

Special Examination in Economics, Thursday, May 15, 1913.
General Examination passed May 19, 1910.
Academic History: Vanderbilt University, 1900-07; Harvard Graduate School, 1907-10, 1911-12; A.B. Vanderbilt, 1906; A.M., Harvard, 1908. Austin Teaching Fellow, 1909-10, 1911-12; Instructor in Economics, 1912-13.
General Subjects: 1. Economic Theory and its History. 2. Economic History since 1750. 3. Statistics. 4. Money, Banking, and Industrial Organization. 5. Transportation and Foreign Commerce. 6. History of American Institutions.
Special Subject: Railroad Transportation.
Committee: Professors Ripley (chairman), Carver, Sprague, and Dr. Copeland.
Thesis Subject: “The History of the Anthracite Coal Industry, with especial reference to the Development of Combination.” (With Professor Ripley.)
Committee on Thesis: Professors Ripley, Taussig, and Sprague.

Joseph Stancliffe Davis.

Special Examination in Economics. Friday, May 16, 1913.
General Examination passed May 17, 1909.
Academic History: Harvard College, 1904-08; Harvard Graduate School, 1908-12; A.B., 1908. Assistant in Economics, 1908-10, 1911-12; Instructor in Economics and Sociology, Bowdoin College, 1912-13.
General Subjects: 1. Economic Theory and its History. 2. Economic History since 1750. 3. Sociology and Social Progress. 4. Money, Banking, and Industrial Organization. 5. History of American Institutions, especially since 1783. 6. Anthropology, especially Ethnology.
Special Subject: Business Corporations, with especial Reference to the Development of Corporate Enterprise in the United States.
Committee: Professors Bullock (chairman), Ripley, Carver, and Schaub.
Thesis Subject: “Corporations in the American Colonies.” (With Professor Bullock.)
Committee on Thesis: Professors Bullock, Channing, and Taussig.

Ralph Emerson Heilman.

Special Examination in Economics (Social Ethics), Monday, May 19, 1913.
General Examination passed May 11, 1911.
Academic History: Morningside College, 1903-06; Northwestern University, 1906-07; Harvard Graduate School, 1909-13; Ph.B., Morningside, 1906; A.M., Northwestern, 1907. Instructor in Economics, 1912-13.
General Subjects: 1. Ethical Theory. 2. Economic Theory and its History. 3. Poor Relief. 4. Social Reform. 5. Sociology. 6. Labor Problems.
Special Subject: The Control of Municipal Public Service Corporations.
Committee: Professors Taussig (chairman), Ripley, Sprague, and Dr. Copeland.
Thesis Subject: “Chicago Traction—A Study in the Efforts of the City to Secure Good Service.” (With Professor Taussig.)
Committee on Thesis: Professors Taussig, Ripley, and Munro.

Rufus Stickney Tucker.

General Examination in Economics, Wednesday, May 28, 1913.
Committee: Professors Bullock (chairman), Turner, Ripley, Sprague, and Dr. Gray.
Academic History: Harvard College, 1907-11; Harvard Graduate School, 1911-13. A.B., 1911; A.M., 1912.
General Subjects: 1. Economic Theory and its History. 2. Statistics. 3. Money and Banking. 4. Economic History since 1750. 5. History of American Institutions. 6. Public Finance.
Special Subject: Public Finance.
Thesis Subject: “The Incidence of Real Estate Taxation.” (With Professor Bullock.)

 

Source: Harvard University Archives. Harvard University, Examinations for the Ph.D. (HUC 7000.70), Folder “Examinations for the Ph.D., 1912-13”.

 

Image Source: Harvard University, card catalogue in Widener Library (ca 1915). Library of Congress Prints and Photographs Division Washington, D.C.

 

Categories
Fields Harvard

Harvard. Seven Ph.D. Examinees in Economics, 1911-12

 

 

For seven Harvard economics Ph.D. candidates this posting provides information about their respective academic backgrounds, the six subjects of their general examinations along with the names of the examiners, the subject of their special subject, thesis subject and advisor(s) (where available).

________________________________________

 

DIVISION OF HISTORY AND POLITICAL SCIENCE
EXAMINATIONS FOR THE DEGREE OF PH.D.
1911-12

Notice of hour and place will be sent out three days in advance of each examination.
The hour will ordinarily be 4 p.m.

 

Wilfred Eldred.

General Examination in Economics, Monday, April 29, 1912.
Committee: Professors Taussig (chairman), Turner, Bullock, Ripley, and Dr. Day.
Academic History: Washington and Lee University, 1906-09; Harvard Graduate School, 1910-12; A.B. Washington and Lee, 1909; A. M., ibid., 1909; A.M., Harvard, 1911.
General Subjects: 1. Economic Theory and its History. 2. Economic History since 1750. 3. Public Finance and Financial History. 4. Money, Banking, and Commercial Crises. 5. Transportation and Foreign Commerce. 6. History of American Institutions.
Special Subject: Economic History of the United States.
Thesis Subject: “Grain Trade and Grain Production since 1860.” (With Professor Carver.)

 

Melvin Chauncey Hunt.

General Examination in Economics (Social Ethics), Tuesday, April 30, 1912.
Committee: Professors Peabody (chairman), Ripley, Carver, Dr. Brackett, and Dr. Rappard.
Academic History: Nebraska Wesleyan University, 1902-06; Boston University, 1907-10; Harvard Graduate School, 1910-12. A.B., Nebraska Wesleyan, 1906; S.T.B., Boston University, 1910.
General Subjects: 1. Economic Theory. 2. Ethical Theory. 3. Poor Relief. 4. Social Reforms. 5. Sociology. 6. Labor Problems.
Special Subject: (undecided).
Thesis Subject: “A Study of the Middle Class Home.” (With Professor Peabody.)

 

Yamato Ichihashi.

General Examination in Economics, Wednesday, May 1, 1912.
Committee: Professors Ripley (chairman), Taussig, Carver, Dr. Rozzer, and Dr. Day.
Academic History: Leland Stanford Junior University, 1904-08; Harvard Graduate School, 1910-12. A.B., Leland Stanford, 1907; A.M., ibid., 1908. Assistant in Economics, Leland Stanford, 1908-10.
General Subjects: 1. Economic Theory and its History. 2. Economic History since 1750. 3. Sociology and Social Reforms. 4. Statistics. 5. Anthropology. 6. Labor Problems and Industrial Organization.
Special Subject: Labor Problems.
Thesis Subject: “Emigration from Japan, and Japanese Immigration into the State of California.” (With Professor Ripley.)

 

Philip Benjamin Kennedy.

General Examination in Economics, Thursday, May 2, 1912.
Committee: Professors Ripley (chairman), Gay, Carver, and Drs. Day and Holcombe.
Academic History: Beloit College, 1900-02, 1903-05; Occidental College, 1905-06; Harvard Graduate School, 1909-12. A.B., Beloit, 1905; Litt.B., Occidental, 1906; A.M., Harvard, 1911.
General Subjects: 1. Economic Theory and its History. 2. Modern Economic History. 3. Sociology and Social Problems. 4. American Institutions. 5. Money and Banking. 6. Transportation and Corporations.
Special Subject: Transportation and Corporations.
Thesis Subject: “Railroad Valuation.” (With Professor Ripley.)

 

Selden Osgood Martin.

Special Examination in Economics, Wednesday, May 8, 1912.
General Examination passed December 1, 1905.
Academic History: Bowdoin College, 1896-97, 1900-03; Harvard Graduate School, 1903-07. A.B., Bowdoin, 1903; A.M., Harvard, 1904. Frederick Sheldon Travelling Fellow, 1910-11. Instructor in the Graduate School of Business Administration, 1910-.
General Subjects: 1. Economic Theory and its History. 2. Statistics. 3. Economic History of the United States. 4. Money, Banking, and Financial History. 5. American History. 6. Constitutional History of England to the Sixteenth Century.
Special Subject: Economic History of the United States.
Committee: Professors Taussig (chairman), Turner, Gay, and Sprague.
Thesis Subject: “Recent Water-Power Development in the United States.”
Committee on Thesis: Professors Taussig, Gay, and Clifford.

 

Alfred Burpee Balcom.

Special Examination in Economics, Friday, May 10, 1912.
General Examination passed May 1, 1911.
Academic History: Acadia College, 1904-07; Harvard Graduate School, 1908-12. S.B., Acadia, 1907; A.M., Harvard, 1909. Austin Teaching Fellow in Economics, 1910-12.
General Subjects: 1. Economic Theory and its History. 2. Economic History since 1750. 3. Sociology and Social Reform. 4. Public Finance and Financial History. 5. Labor Problems and Industrial Organization. 6. Philosophy.
Special Subject: Economic Theory in England from Adam Smith to the present time.
Committee: Professors Taussig (chairman), Bullock, Carver, Sprague, and Dr. Rappard.
Thesis Subject: “The Development of the English Poor Law Policy.” (With Professor Taussig.)
Committee on Thesis: Professors Taussig, Bullock, and Carver.

 

Ralph Emerson Heilman.

Special Examination in Economics (Social Ethics), Friday, May 17, 1912.
General Examination passed May 11, 1911.
Academic History: Morningside College, 1903-06; Northwestern University, 1906-07; Harvard Graduate School, 1909-12; Ph.B., Morningside, 1906; A.M., Northwestern, 1907.
General Subjects: 1. Ethical Theory. 2. Economic Theory and its History. 3. Poor Relief. 4. Social Reforms. 5. Sociology. 6. Labor Problems.
Special Subject: The Control of Municipal Public Service Corporations.
Committee: Professors Taussig (chairman), Peabody, Munro, and Dr. Holcombe.
Thesis Subject: “Chicago Traction.” (With Professor Taussig.)
Committee on Thesis: Professors Taussig, Ripley, and Munro.

 

Source: Harvard University Archives. Harvard University, Examinations for the Ph.D. (HUC 7000.70), Folder “Examinations for the Ph.D., 1911-12”.

Image Source: John Harvard Statue (1904). Library of Congress. Photos, Prints and Drawings.

 

Categories
Fields Harvard

Harvard. Ph.D. candidates examined 1910-11

 

 

This posting provides information for four Harvard economics Ph.D. candidates: their respective academic backgrounds, the six subjects of their general examinations along with the names of the examiners, the subject of their special subject, thesis subject and advisor(s) (where available).

________________________________________

 

DIVISION OF HISTORY AND POLITICAL SCIENCE
EXAMINATIONS FOR THE DEGREE OF PH.D.
1910-11

Notice of hour and place will be sent out three days in advance of each examination.
The hour will ordinarily be 4 p.m.

Alfred Burpee Balcom.

General Examination in Economics, Monday, May 1, 1911.
Committee: Professors Taussig (chairman), Bullock, Carver, Sprague, Young, and Perry.
Academic History: Acadia College, 1904-07; Harvard Graduate School, 1908-11. S.B., Acadia, 1907; A. M., Harvard, 1909. Austin Teaching Fellow, 1910-11.
General Subjects: 1. Economic Theory and its History. 2. Economic History since 1750. 3. Sociology and Social Reform. 4. Public Finance and Financial History. 5. Labor Problems and Industrial Organization. 6. Philosophy.
Special Subject: Economic Theory.
Thesis Subject: “Nassau William Senior as an Economist.” (With Professor Taussig.)

Lucius Moody Bristol.

General Examination in Economics (Social Ethics), Thursday, May 4, 1911.
Committee: Professors Peabody (chairman), Taussig, Carver, Sprague, Young, and Dr. Brackett.
Academic History: University of North Carolina, 1894-95; Boston University School of Theology, 1896-99; Harvard Divinity School, 1909-10; Harvard Graduate School, 1910-11. A.B., North Carolina, 1895; S.T.B., Boston University, 1899.
General Subjects: 1. Ethical Theory. 2. Economic Theory. 3. Labor Problems. 4. Social Reforms. 5. Sociology. 6. Statistics.
Special Subject: Social Reform.
Thesis Subject: “Conservation of Vital Forces in Boston.” (With Professor Peabody.)

Johann Gottfried Ohsol.

General Examination in Economics, Friday, May 5, 1911.
Committee: Professors Gay (chairman), Bullock, Carver, Sprague, Dr. Foerster, and Dr. Holcombe.
Academic History: Polytechnic Institute of Riga, 1899-1903; Harvard Graduate School, 1909-11. Candidate in Commerce, Riga, 1903.
General Subjects: 1. Economic Theory and its History. 2. Economic History since 1750. 3. Sociology and Social Reform. 4. Public Finance and Financial History. 5. Labor Problems and Industrial Organization. 6. History of American Institutions.
Special Subject: Labor Problems.
Thesis Subject: (undecided).

Ralph Emerson Heilman.

General Examination in Economics (Social Ethics), Thursday, May 11, 1911.
Committee: Professors Peabody (chairman), Taussig, Bullock, Carver, Dr. Brackett and Dr. McConnell.
Academic History: Morningside College, 1903-06; Northwestern University, 1906-07; Harvard Graduate School, 1909-11. Ph.B., Morningside, 1906; A.M., Northwestern, 1907.
General Subjects: 1. Ethical Theory. 2. Economic Theory and its History. 3. Poor Relief. 4. Social Reforms. 5. Sociology. 6. Labor Problems.
Special Subject: (undecided).
Thesis Subject: “Chicago Traction.” (With Professor Ripley.)

 

Source: Harvard University Archives. Harvard University, Examinations for the Ph.D. (HUC 7000.70), Folder “Examinations for the Ph.D., 1910-11”.

Image Source: Widener Library, 1915. Library of Congress Prints and Photographs Division, Washington, D.C. Digital ID:  cph 3c14486

Categories
Courses Exam Questions Harvard

Harvard. Final Exams in Economics. 1913-14.

 

 

This posting merges information from three sources: brief course descriptions from the annual course announcement published for the Division of History, Government and Economics for the academic year 1913-14 in the Harvard Register; final examination questions published by Harvard in June 1914; and the mid-year (i.e. February) examination questions for two courses taught by Frank Taussig and pasted in a file scrapbook containing what appears to be all of his Harvard examinations.

At hathitrust.org there are online copies of the annual June publication of examination questions for 1912-13 through 1915-16. A transcription of the 1912-13 economics examinations has been posted earlier.

While sixteen courses have published  final examinations that are transcribed below, there were still some seven or so economics courses not included in the published June volume. Further the mid-year (i.e. February) final exams for year long courses were not included in the published collection.

________________________________

Principles of Economics

Course Description
Economics A

[Economics] A. (formerly 1). Principles of Economics. Tu., Th., Sat., at 11.

Professor TAUSSIG and Asst. Professor DAY, assisted by Messrs. Burbank, J. S. Davis, R. E. Heilman, and others.

Course A gives a general introduction to economic study, and a general view of Economics for those who have not further time to give to the subject. It undertakes a consideration of the principles of production, distribution, exchange, money, banking, international trade, and taxation. The relations of labor and capital, the present organization of industry, and the recent currency legislation of the United States will be treated in outline.

The course will be conducted partly by lectures, partly by oral discussion in sections. A course of reading will be laid down, and weekly written exercises will test the work of students in following systematically and continuously the lectures and the prescribed reading. Course A may not be taken by Freshmen without the consent of the instructor.

 

Mid-Year Exam
Economics A

Arrange your answers strictly in in the order of the questions. Answer all the questions.

  1. State concisely the distinctions between the following (omit one): —

(a) free goods and public goods;
(b) saving, investment, the creation of capital;
(c) subsidiary coinage and limping standard;
(d) industrial crisis and financial panic;
(e) deposits in commercial banks and deposits in savings banks.

  1. Which among these distinctions is important for the understanding of the following, and wherein? (Omit one.)

(a) the influence of credit on prices;
(b) the benefits to be expected from a centralized banking system;
(c) the rates which a municipality charges for water supplied to consumers;
(d) the effects of public borrowing (government debts);
(e) silver certificates.

  1. (a) Suppose a great and lasting increase in the demand for skates: what would you expect to be the immediate, what the ultimate effects on the value of skates?
    (b) Suppose a great and lasting increase in the demand for Indian corn: what would you expect to be the immediate, what the ultimate effects on the value of Indian corn?
    (c) Suppose a great and lasting increase in the demand for wheat straw: what would you expect to be the immediate, what the ultimate effects on the value of wheat?
  2. “Here cost is supposed to be uniform but not constant, — it becomes less per unit as the number of units increases.” Explain the terms “uniform” and “constant,” and the conditions of production described in the extract. How is value determined under these conditions (illustrate either by diagram or by example)?
  3. In which direction and by what process would the following tend to affect the price to the consumer in the United States of a bushel of wheat: (1) adoption of bimetallism by the United States at the ratio of 16 to 1; (2) development of organized speculation; (3) a successful corner in wheat?
  4. Explain: —

Central Reserve City Bank;
Federal Reserve Bank;
U.S. Treasury Gold Reserve;
Bank of England Reserve.

  1. Suppose the people of one country to lend, through a long period, large sums annually to the people of another country; trace the effects in the lending country, immediate and ultimate, on

the flow of specie;
merchandise imports and exports;
the price of foreign exchange.

Would you expect such a lending country to have a “favorable” or an “unfavorable” balance of trade?

  1. Suppose the following course of prices: —

 

Price of silver
per oz.
Price of wheat
per bushel
Index numbers of general prices
1873 $1.30 $1.32 130
1895 0.65 0.67 80
1912 0.61 1.10 110

Would the figures indicate that the value of silver changed between 1873 and 1895? The value of gold? of wheat?

Would they indicate that the value of silver changed from 1895 to 1912? of gold? of wheat?

 

Final Exam
Economics A

  1. Arrange the following items in the form of a bank statement showing in parallel columns the liabilities and resources: —

Real estate, $30,000; Surplus, $30,000; Deposits, $283,000; Loans, $300,000; Reserve, $65,000; Undivided profits, $12,000; Other assets, $10,000; Capital stock, $100,000; Bonds and stocks, $80,000; Notes, $75,000; Due from banks, $15,000.

Draw up a similar statement showing condition after each of the following operations: —

(a) The bank makes a new loan of $1000 for 3 months at the discount rate of 4% per annum. Proceeds are taken 1/3 in specie, 1/3 in the bank’s own notes, and the balance in a deposit account.

(b) The bank adds $5000 to its surplus, and declares a dividend of 2%. Stockholders take half of the dividend in gold, and leave half on deposit with the bank.

  1. What would be the immediate effect, what the ultimate effect, of a large increase in the supply of money on (a) money wages, (b) real wages, (c) business profits, (d) the bank rate of discount?
  2. “The principle of protection is to build up our home industries by manufacturing our own products. This gives our people employment, keeps the money in the country, and makes this country an independent and self-reliant nation.”

Wherein are these arguments valid? Wherein invalid? Give your reasons.

  1. “The outcome of the discussion of demand and supply (with reference to capital and interest) can be stated in simple form under the theory of value. The several installments of savings can be had at various rates, some for a small reward, some for a larger reward. The case is thus one of varying supply price, coming under the principle of increasing costs.”

Explain, and illustrate by diagram.

  1. “The effect of high prices for land and high rents is apparent. Industries will be slow to locate in Pittsburgh if rents or prices of land are higher than in other cities. A higher rent or interest on higher-price of land bought for building, will be a constant added charge on cost of operation. Consequently, industries will tend to shun a city where this higher cost is incurred.” Do you think this consequence will ensue?

Suppose a tax in this city (not levied in other cities) on the future increase of land values; would industries shun the city?

  1. Explain wherein the problems would be different in fixing minimum wages (a) for common unskilled labor, (b) for various grades of skilled labor, (c) for women.
  2. How great has been the development of coöperation in production? What explanation can you give?

What is the ground for saying that “maturity” makes an industry more proper for public management?

“The inevitable attitude of the hired workman is to favor arrangements that seem to make work and to oppose those that seem to lessen work.”

Why should this attitude be thought “inevitable”?

  1. Explain, and give in each case, if possible, an illustration drawn from American or British experience in the taxation of land:

Increment tax;
Stoppage at the source;
Incidence of a tax;
Progressive tax.

 

________________________________

 

Statistics

Course Description
Economics 1

[Economics] 1 1hf. Statistics. Half-course (first half-year). Mon., Wed., Fri., at 11. Asst. Professor DAY.

This course will deal primarily with the elements of statistical method. The following subjects will be considered: methods of collecting and tabulating data; the construction and use of diagrams; the use and value of the various types and averages; index-numbers; dispersion; interpolation; correlation. Special attention will be given to the accuracy of statistical material.

In the course of this study of statistical method, examples of the best statistical information will be presented, and the best sources will be indicated. Population and vital statistics will be examined in some measure, but economic statistics will predominate.

Open only to those who, having passed satisfactorily in Economics A, secure the consent of the instructor.

 

Final Exam
Economics 1

  1. Indicate two methods of correcting death-rates for age- and sex-distribution.
  2. What are the different methods of collecting workmen’s budgets? What are the advantages and disadvantages of each of these methods?
  3. What are the chief difficulties encountered in the use of statistics of imports and exports?
  4. Compare the advantages and disadvantages of the mode and arithmetic average as statistical types.
  5. Describe and criticise the different methods of presenting wage statistics. Cite instances of the use of each.
  6. Define correlation. What is Pearson’s coefficient of correlation? Indicate its use and interpretation.
  7. Explain briefly: ogive; lag; probable error; Galton graph; standard deviation; logarithmic curve; ratio of variation; Lorenz curve.

________________________________

 

European Industry and Commerce in the Nineteenth Century

Course Description
Economics 2a

[Economics] 2a 1hf. European Industry and Commerce in the Nineteenth Century. Half-course (first half-year). Tu., Th., Sat., at 9. Professor GAY, assisted by —.

Course 2a undertakes to present the general outlines of the economic history of western Europe since the Industrial Revolution. Such topics as the following will be discussed: the economic aspects of the French Revolution and the Napoleonic régime, the Stein-Hardenberg reforms, the Zoll-Verein, Cobden and free trade in England, labor legislation and social reform, nationalism and the recrudescence of protectionism, railways and waterways, the effects of transoceanic competition, the rise of industrial Germany.

Since attention will be directed in this course to those phases of the subject which are related to the economic history of the United States, it may be taken usefully before Economics 2b.

 

Final Exam
Economics 2a

  1. When did the Industrial Revolution take place in Germany? Why did it come later there than in England? In how far was it brought about by analogous causes?
  2. Compare the scale of production and specialization in the cotton, shoe, and wool manufacturing industries in England and France. Give reasons for contrasts.
  3. Discuss the part which the banks have played in the promotion of industrial concentration in the electrical, chemical, and mining industries in Germany. What other factors have encouraged the development of these industries.
  4. (a) Account for the relatively high capitalization of the railways in England.
    (b) How has the “cost of service” principle been applied in the fixing of freight rates on the Prussian railways?
  5. What have been the periods of prosperity in English agriculture in the nineteenth century? And what have been the causes? How have these periods of prosperity affected the agricultural laborer?
  6. What interests have supported the recent tariff reform movement in England? Why? Do you think that from the English standpoint such a change in policy is desirable? Why or why not?

________________________________

 

Economic and Financial History of the United States

Course Description
Economics 2b

[Economics] 2b 2hf. Economic and Financial History of the United States. Half-course (second half-year). Tu., Th., Sat., at 9. Professor GAY, assisted by —.

The following are among the subjects considered: aspects of the Revolution and commercial relations during the Confederation and the European wars; the history of the protective tariff policy and the growth of manufacturing industries; the settlement of the West and the history of transportation, including the early canal and turnpike enterprises of the states, the various phases of railway building and the establishment of public regulation of railways; banking and currency experiences; various aspects of agrarian history, such as the public land policy, the growth of foreign demand for American produce and the subsequent competition of other sources of supply; certain social topics, such as slavery and its economic basis, and the effects of immigration.

 

Final Exam
Economics 2b

  1. Discuss the bearing of the mercantile theory upon American commercial history before 1860.
  2. Comment on the following statements by William McKinley:

(a) “A low tariff or no tariff has always increased the importation of foreign goods until our money ran out; multiplied our foreign obligations; produced a balance of trade against our country; supplanted the domestic producer and manufacturer; impaired the farmer’s home market without improving his foreign market; decreased the industries of the nation; diminished the value of nearly all our property and investments and robbed labor of its just rewards. This is the verdict of our history.”

(b) “Periods of low tariff synchronize with industrial depression ” [in American history].

  1. “In the twenty years [after 1816] institutions were arising and changing, and centers of social gravity shifting. It was essentially a time of realignment of interests.”

State your grounds of agreement or disagreement with this view, and compare these changes with those in the period since 1890.

  1. Illustrate with three examples the problem of localization of industry in the United States.
  2. “The Civil War was won by the McCormick reaper.” How far was this true, and why?
  3. Write briefly on the following topics: —

(a) The competition between anthracite and coke in the iron industry.
(b) Willoughby’s estimate of the future of integration in industry.

________________________________

 

Money, Banking, and Commercial Crises

Course Description
Economics 3

[Economics] 3. Money, Banking, and Commercial Crises. Mon., Wed., Fri., at 1.30. Asst. Professor DAY, assisted by —.

This course aims to analyze the principal problems of money and credit. An examination is first made of the more important existing monetary systems. This is followed by a careful review of the more instructive chapters in the monetary history of England, Germany, France, the United States, Austria, British India, Mexico, and the Philippines.

The nature, origin, and early growth of commercial banking are considered. An investigation of present banking practice in England, France, Germany, and Canada is followed by a study of banking history and present banking problems in the United States. In this connection foreign exchange and the money markets of London, Paris, Berlin, and New York are examined.

Finally attention is turned to those problems of money and credit which appear most prominently in connection with economic crises. Though emphasis is thrown upon the financial aspects of the trade cycle, the investigation covers the more fundamental factors causing commercial and industrial fluctuations.

Short papers upon assigned topics will be required of all students.

 

Final Exam
Economics3

  1. Suppose the United States were to permit the free coinage of our present silver dollar. How would this tend to affect the (1) monetary stock of the United States; (2) mint price of silver; (3) value of the dime; (4) price of gold jewelry; (5) value of gold certificates; (6) prices in England; (7) balance of international payments; (8) rates of foreign exchange? Give explanations throughout.
  2. How is the value of irredeemable paper money to be measured? What determines the value of such money? What are the most important questions in the resumption of specie payments after a period of irredeemable paper? If possible, illustrate your points from the experience of the United States.
  3. Define discount market. Describe the English discount market. How has the absence of such a market affected banking in the United States? What provisions of the Federal Reserve Act are designed to develop a discount market in this country?
  4. How and why have panics and crises in the United States tended to affect (1) aggregate bank loans; (2) reserves of the national banks; (3) amount of bank notes in circulation; (4) quotations of stocks and bonds on the New York Stock Exchange; (5) rates of foreign exchange in New York?
  5. Briefly describe the following phenomena in the panic of 1907; (1) currency premium; (2) hoarding; (3) the domestic exchanges; (4) substitutes for cash.
  6. By what means and to what extent, if at all, does the Federal Reserve Act provide for an effective centralized control of credit in the United States?

________________________________

 

Economics of TransportationCourse Description
Economics 4a

[Economics] 4a 1hf. Economics of Transportation. Half-course (first half-year). Tu., Th., Sat., at 11. Professor RIPLEY, assisted by —.

A brief outline of the historical development of rail and water transportation in the United States will be followed by a description of the condition of transportation systems at the present time. The four main subdivisions of rates and rate-making, finance, traffic operation, and legislation will be considered in turn. The first deals with the relation of the railroad to shippers, comprehending an analysis of the theory and practice of rate-making. An outline will be given of the nature of railroad securities, the principles of capitalization, and the interpretation of railroad accounts. Railroad operation will deal with the practical problems of the traffic department, such as the collection and interpretation of statistics of operation, pro-rating, the apportionment of cost, depreciation and maintenance, etc. Under legislation, the course of state regulation and control in the United States and Europe will be traced.

 

Final Exam
Economics 4a1

  1. Railroad A. is capitalized at $50,000 per mile, — $35,000 in five per cent bonds and the rest in stock. Railroad A. earns about $2500 net per mile. Railroad B. earns about $4000 net per mile on a capitalization of $90,000 per mile, — $50,000 in four per cent bonds, the balance in stock. Which is the stronger road financially? What about the relative ability of the two roads to give service at low rates?
  2. Describe the general plan by which competition in Trunk Line territory was eliminated within the last decade. What has since happened?
  3. What has been in general the course of prices of railway securities since 1890? Briefly state the causes.
  4. What was the final plan adopted for dissolution of the Union-Southern Pacific combination?
  5. How was the question of land valuation for railroad purposes in the Minnesota Rate Case treated?
  6. What is the gist of the Fourteenth Amendment to the Constitution of the United States? Merely name a few of the most important cases applying it to railroads since 1870, and in a sentence in each case outline the point covered.
  7. Outline a typical case, real or hypothetical, showing how Federal and State authority may come in conflict in the matter of rate-making.
  8. When and why was the Commercial Court created? Outline the result of the experiment.
  9. It has been urged that railroad monopoly under adequate Government regulation may serve the public as well as competition. Do you agree with this view? State your reasons and cite instances.

________________________________

 

Economics of Corporations

Course Description
Economics 4b

[Economics] 4b 2hf. Economics of Corporations. Half-course (second half-year). Tu., Th., Sat., at 11. Professor RIPLEY, assisted by —.

This course will treat of the fiscal and industrial organization of capital, especially in the corporate form. The principal topic considered will be industrial combination and the so-called trust problem. This will be broadly discussed, with comparative study of conditions in the United States and Europe. The development of corporate enterprise, promotion, and financing, accounting, liability of directors and underwriters, will be described, not in their legal but in their economic aspects; and the effects of industrial combination upon efficiency, profits, wages, prices, the development of export trade, and international competition will be considered in turn.

 

Final Exam
Economics  4b

Answer in order — omitting any one question.

  1. What are the principal advantages of a stable rate of dividends? What influences tend to cause departure therefrom?
  2. Outline two ways at least of securing temporary relief by appeal to stock-holders in case of threatened insolvency of a corporation.
  3. What is the most important economy incident to production under monopoly of the market, as distinct from mere large-scale production?
  4. Why is the financial experience of the American Mercantile Marine Company significant?
  5. Outline the course of enforcement of the Sherman Act. How largely did underlying economic causes, as distinct from purely personal ones, play a part?
  6. Outline the device, in case of corporate promotion, for making an issue of stock full-paid in order to relieve investors against further assessments.
  7. Would price regulation — as by the American Publishers Association — fixing the retail price of books and excluding cut-rate dealers from supplies, seem to be debarred by the Standard Oil decision?
  8. Are financial abuses such as an excessive issue of securities as characteristic of German industrial combinations as of those in the United States?
  9. Contrast price fixing by law for monopolized commodities with the regulation of railroad rates. How may such an issue arise in connection with amendment of the Sherman Act?

________________________________

Public Finance

Course Description
Economics 5

[Economics] 5. Public Finance, including the Theory and Methods of Taxation. Mon., Wed., Fri., at 9. Professor BULLOCK.

This course covers the entire field of public finance, but emphasizes the subject of taxation. After a brief survey of the history of finance, attention is given to public expenditures, commercial revenues, administrative revenues, and taxation, with consideration both of theory and of the practice of various countries. Public credit is then studied, and financial legislation and administration are briefly treated.

Systematic reading is prescribed, and most of the exercises are conducted by the method of informal discussion. Candidates for distinction will be given an opportunity to write theses.

Graduate students are advised to elect Economics 31.

 

Final Exam
Economics 5

  1. Discuss the different definitions of a tax.
  2. Discuss Adam Smith’s maxims of taxation.
  3. Discuss the incidence of an exclusive tax on land.
  4. Discuss the incidence of taxes upon mortgages in the United States.
  5. Compare the working of the general property tax in the United States with its working in Switzerland.
  6. Discuss the proposition that income is the normal source of taxation.
  7. Discuss the leading arguments for and against progressive taxation.
  8. Discuss the leading arguments of Shearman and Seligman for and against the single tax.

________________________________

 

Trade Unionism and Allied Problems

Course Description
Economics 6a

[Economics] 6a 1hf. Trade Unionism and Allied Problems. Half-course (first half-year). Tu., Th., Sat., at 10. Professor RIPLEY, assisted by —.

This course will deal mainly with the economic and social relations of employer and employed. Among the topics included will be: the history of unionism; the policies of trade unions respecting wages, machinery, output, etc.; collective bargaining; strikes; employers’ liability and workmen’s compensation; efficiency management; unemployment, etc., in the relation to unionism, will be considered.

Each student will make at least one report upon a labor union or an important strike, from the original documents. Two lectures a week, with one recitation, will be the usual practice.

 

Final Exam
Economics 6a

  1. Outline the principal phases of development of organized labor in the United States, with especial reference to conditions at the present time. In conclusion name five or six of the most significant events which define the present situation.
  2. What are the three most essential features of a collective bargain between workmen and employers?
  3. What is the feature in common of all minimum wage laws, as in Victoria and of compulsory arbitration statutes like those of New Zealand? Wherein does the policy differ most profoundly from ours?
  4. Name in a sentence in each of as many of the following cases as possible, the essential point at issue.

(a) The Danbury hatters.
(b) Allen v. Flood.
(c) New York Bakeshop law.
(d) Bucks Stove Co. case.
(e) Taff Vale Railway.
(f) Holden v. Hardy. (Utah.)

  1. How, other than by incorporation, is a greater measure of legal responsibility of trade unions to be attained?
  2. Discuss scientific management from the viewpoint of organized labor.
  3. What is the significant feature of the new type of state labor bureau, like the Wisconsin Industrial Commission?
  4. Compare the present legal status of the non-union man in England and the United States.

________________________________

 

Theories of Distribution and Distributive Justice

Course Description
Economics 7

[Economics] 7. Theories of Distribution and Distributive Justice. Tu., Th., Sat., at 10. Professor CARVER and an assistant.

Course 7 undertakes an analysis of the laws of value, as applied to consumable goods and to agents of production, including labor, land, capital, and management; the laws determining wages, rent, interest, and profits; and an examination of the relation of the laws of value to the problem of social adjustment; the social utility of various forms of property; also a critical reading of various works on the distribution of wealth, on socialism, on the single tax, and other special schemes for attaining the ideals of economic justice.

 

Final Exam
Economics 7

  1. What have you read for this course during the year? What parts of the reading interested you most? What parts interested you least? What parts gave you most difficulty?
  2. State and criticise in detail Fisher’s theory of the value of money.
  3. State and criticise Laughlin’s theory of the value of money.
  4. A well-secured note of a good corporation for $100 has four years to run. It pays 7 per cent interest. It is taxed at 1 per cent. The prevailing rate of interest on such paper is 5 per cent. What is the note worth?
  5. What is your own theory of crises?
  6. A law requiring proprietors of saw-mills to insure their workmen against accident would lead to increased cost of production, and higher prices, for lumber. Would a law requiring all employers similarly to insure lead to higher prices all around? Why or why not?
  7. What do you think of the single-tax contention that all taxes except land-taxes are burdens on industry, and restrict production?
  8. Summarize and criticise Shearman’s arguments for the single tax.
  9. State and criticise Clark’s argument to prove that ” unearned increments ” in land values off-set depreciation on buildings, and so increase the amount of building.

________________________________

 

Principles of Sociology

Course Description
Economics 8

[Economics] 8. Principles of Sociology. — Theories of Social Progress. Mon., Wed., Fri., at 10. Professor CARVER and an assistant.

An analytical study of social life and of the factors and forces which hold society together and give it an orderly development. The leading social institutions will also be studied with a view to finding out their relation to social well-being and progress.

The reading will be selected from various writers who have treated the problems of human progress and social adjustment.

Course 8 is open only to students who have passed in Economics 1.

 

Final Exam
Economics 8

Sociology

  1. Make a two-page topical outline of the course as a whole.
  2. What topics in the course would you wish to have treated more fully? What topics seemed to you to have proportionately too much attention? What parts of the reading interested you most? What parts of the reading did you find most helpful? What parts of the reading gave you most difficulty? What parts of the reading would you prefer to see omitted?
  3. In what respects does the imitation theory fall short of an adequate social psychology?
  4. Discuss the economic interpretation of history.
  5. Discuss the “color line.”
  6. Summarize Spencer’s theory of the origin of religion. In what respects is it deficient?
  7. To what does Giddings attribute the rise of democracy? In what ways does he think that democracy changes the functions of government?
  8. State and illustrate Giddings’ “three stages of civilization.” Compare this conception with the rival views of Hegel, Comte and Spencer.
  9. Summarize John Dewey’s “Interpretation of Savage Mind.”
  10. Summarize the theory of progress developed in the lectures. What is your own view?

________________________________

 

Principles of Accounting

Course Description
Economics 9

[Economics] 9. Principles of Accounting. Mon., Wed., Fri., at 11. Associate Professor Cole, assisted by Messrs. and —.

This course is designed to show the processes by which the earnings and values of business properties are computed. It is not intended primarily to afford practice in book-keeping; but since intelligent construction and interpretation of accounts is impossible without a knowledge of certain main types of book-keeping, practice sufficient to give the student familiarity with elementary technique will form an important part of the work of the course. The chief work, however, will be a study of the principles that underlie the determination of profit, cost, and valuation. These will be considered as they appear in several types of business enterprise. Published accounts of corporations will be examined, and practice in interpretation will be afforded. The instruction will be chiefly by assigned readings, discussions, and written work.

Course 9 is not open to students before their last year of undergraduate work. For men completing their work at the end of the first half-year, it may be counted, with the consent of the instructor, as a half-course. It is regularly open only to Seniors and to Graduates who have passed in Economics A. Students intending to enter the Graduate School of Business Administration are expected to take this course in preparation for the advanced courses in accounting.

 

Final Exam
Economics 9

PRINCIPLES OF ACCOUNTING
Associate Professor Cole

  1. Illustrate, by imaginary entries, any book from which posting may be made in lump sum not only for many items to be debited to one account, but also for many items to be credited to each of various other accounts. [Show at least three items to be posted in lump sum for each of three accounts, and show at least two items that must be posted individually.]
  2. Two successive condensed balance sheets show the following figures: —

January 1, 1913

Real Estate $50,000 Capital Stock $100,000
Merchandise 75,000 Bills Payable 25,000
Accounts Receivable 30,000 Accounts Payable 30,000
Miscellaneous Assets 7,000 Surplus 7,000
$162,000 $162,000

 

January 1, 1914

Real Estate $53,000 Capital Stock $100,000
Merchandise 77,000 Bills Payable 25,000
Accounts Receivable 12,000 Accounts Payable 20,000
Miscellaneous Assets 7,000 Surplus 7,000
Reserve for Depreciation 5,000
Dividends 7,000
$149,000 $149,000

Assuming that no dividends were paid, what were the profits for the year?
Where are they?

  1. Should you charge against revenue or to capital (giving your reason in each case) the cost of the following : —

(1) An extra wheel, carried ready for emergency, for an automobile truck.
(2) Wages of an extra watchman employed because construction work has removed a part of the wall of a store.
(3) Installation of an automatic sprinkler system required because during a strike fanatics have threatened incendiarism.
(4) Repairs of a building after a slight collapse due to the disintegration of concrete frozen during construction.
(5) Directories, handbooks, encyclopedias, etc., in the office of a professional firm that must keep informed of the latest scientific and professional news.

  1. What is the probable explanation of the following entries?
Good Will $25,000
To Andrew Jackson $25,000
Subscriptions 200,000
To Stock Subscribed 175,000
Premium Surplus 25,000
Cash 50,000
Andrew Jackson 150,000
To Subscriptions 200,000
Stock Subscribed 175,000
To Capital Stock 175,000

 

  1. How should you distribute the following general expenses over the departments of a department store, grouping the expenses as far as feasible: —
Rent,
Light,
Heat,
Insurance,
Taxes,
General Administration,
Correspondence,
Accounting,
Advertising,
Welfare Work.
  1. The estimated wear and tear on machinery in a shop is $12,000 a year. The profits are figured monthly and $1,000 is taken into the cost accounts for wear and tear on the last day of every month. The amount spent (in cash) for repairs and renewals is as follows: February 15, $500; March 15, $1,200; June 15, $2,500; August 15, $8,000; December 15, $1,500. Show the entry or entries for wear and tear for (1) each last day of the month, (2) the five dates given above, (3) closing at the end of the year. [Show either journal or ledger, with dates.]
  2. Bonds are issued to the amount of $12,000,000, payable in twenty-five years, with interest at 5 per cent annually (in semiannual payments). The credit of the issuing company is not good enough to warrant investors in lending on a basis of less than 5½ per cent. The bonds are accordingly sold for $11,190,084.90. Where will the discount appear on the issuer’s statements — income sheet, balance sheet, both, neither? If either or both, how and where?

Bond tables give the value of such bonds six months later as $11,197,812.23. When the first interest, of $300,000, is paid, what entry or entries should be made? Write the explanation portion of such entries.

  1. Suppose that the cost accounts of a manufacturing business are carried through the general ledger, and that the accounts have been closed so far as to show on the ledger all the figures for the operating statement. This statement is as follows: —

Operating statement, May 1, 1913, to April 30, 1914

Sales $297,000
Raw materials on hand, 5/1/13 $26,000
Raw materials bought 107,000
Raw materials handled 133,000
Raw materials on hand, 4/30/14 18,000
Raw materials consumed 115,000
Wages paid $54,000
Less balance due, 5/1/13 2,000
52,000
Wages due, 4/30/14 900
Wages cost 52,900
Taxes 1,500
Interest prepaid, 5/1/13 600
Interest paid in and for year 1,000 1,600
General manufacturing expenses 30,000
Manufacturing cost 201,000
Goods in process, 5/1/13 10,000
Cost of goods for year 211,000
Goods in process, 4/30/14 7,000
Cost of goods finished in year 204,000
Stock on hand, 5/1/13 60,000
Cost of finished goods handled 264,000
Stock on hand, 4/30/14 20,000
Cost of goods sold 244,000
Selling cost 10,000 254,000
Net profits 43,000

Show the trial balance of ledger totals (not balances) for the cost accounts, supposing that the net balance of all accounts not involved in the cost accounting is $1100 on the credit side.

  1. Below are four columns of a six-column statement which were drawn up for a special purpose (sometimes waiving proper classifications) with the intention of filling out the remaining columns. Fill out the other two columns, and then present a proper form of balance sheet and income sheet (so far as the facts are known to you) for the railroad whose operations are covered by the figures, assuming that dividends of 6 per cent are declared, but not paid, at the end of the year.
Capital Stock 50.0 50.0
Bonded Debt 150.4 150.4
Accounts Receivable 12.5 12.5
Accounts Payable 2.0 2.0
Road and Equipment 101.3 101.3
Investments 102.7 102.7
Cash 14.7 14.7
Supplies 5.7 5.7
Advances 12.5 12.5
Transportation 13.9 46.7 2.5
Maintenance of Way and Structures 5.5 .4 1.2
Maintenance of Equipment 6.8 1.6
Traffic 1.1
General Expense 1.2 .4
Taxes 1.5 3.0
Other Income 6.5
Interest 6.0 1.5
Miscellaneous Expense 4.4 1.9 1.8
Surplus _______ 33.4 ______ 33.4
289.8 289.8 251.3 247.4

 

________________________________

Economic Theory

Course Description
Economics 11

[Economics] 11. Economic Theory. Mon., Wed., Fri., at 2.30. Professor TAUSSIG.

Course 11 is intended to acquaint the student with some of the later developments of economic thought, and at the same time to train him in the critical consideration of economic principles and the analysis of economic conditions. The exercises are accordingly conducted mainly by the discussion of selected passages from the leading writers; and in this discussion the students are expected to take an active part. The writings of J. S. Mill, Cairnes, F. A. Walker, Clark, Marshall, Böhm-Bawerk, and other recent authors, will be taken up. Attention will be given chiefly to the theory of exchange and distribution.

 

Mid-Year Exam
Economics 11

Arrange your answers in the order of the questions. One question may be omitted.

  1. “The distinction, then, between Capital and Not-capital, does not lie in the kind of commodities, but in the mind of the capitalist — in his will to employ them for one purpose rather than other; and all property, however ill adapted in itself for the use of labourers, is a part of capital, so soon as it, or the value to be received from it, is set apart for productive reinvestment. The sum of all the values so destined by their respective possessors composes the capital of the country.”

What is to be said for this doctrine, what against it? By whom was it maintained?

  1. “Prices of commodities in great measure are fixed by supply and demand, but, except temporarily, they cannot be less than all costs, including wages and taxes, entering directly or indirectly into their production and distribution, together with some profit for the use of the capital employed. Hence an increase of the wages or cost of labor usually must be paid by consumers. A general increase of the wages of all labor would cause an equivalent increase of the price of nearly every product of labor and a general increase of the cost of living. The increased wages of the laborers then would not buy more than did their former wages and they would be no better off than before the increase. For this reason the economic welfare of the masses in the aggregate cannot be materially improved by the simple expedient of raising generally the wages of labor.”

What would Ricardo say to this? J. S. Mill? Your own view?

  1. Marx’s doctrine, that value is embodied labor, has been said to be essentially the same as Ricardo’s doctrine that value rests on the labor given to producing an article. Why or why not?
  2. Suppose an increase in the demand for a commodity, in the schedule sense: —

(a) For short periods, under what conditions, if under any, would you expect supply price to rise? to fall?
(b) For long periods, under what conditions, if under any, would you expect supply price to rise? to fall?

Note whether your answer differs in any particular from that to be expected from Marshall.“The part played by the net product at the margin of production in the modern doctrine of distribution is apt to be misunderstood. In particular many able writers have supposed that it represents the marginal use of a thing as governing the value of the whole. It is not so; the doctrine says we must go to the margin to study the action of those forces which govern the value of the whole; and that is a very different affair.”

Explain.

  1. “It has sometimes been argued that if all land were equally advantageous and all were occupied, the income derived from it would not be a true rent, but a monopoly rent.”

Under what conditions, if under any, would there be true rent in such a case? Under what conditions, if under any, would there be a monopoly rent?

  1. “The derived supply price [of one of a group of things having a joint supply price] is found by a rule that it must equal the excess of the supply price for the whole process of production over the sum of the demand prices of all the other joint products.”

Explain, illustrating by diagram.

State the corresponding rule for the derived demand price of one of a group of commodities for which there is a joint demand.

  1. (a) “In hundreds and thousands of suburban homes the question is asked every day, “How much milk shall we take in today, ma’am?” or “How much bread?” and the housewife knows without consideration that if she ordered one loaf of bread and one pint of milk, the marginal significance of bread and milk would be higher than their price, and if she said six loaves and five quarts of milk, the marginal loaf and pint would not be worth their price. Such orders, therefore, never enter into her head. But she deliberates, perhaps, whether she will want three loaves of bread or four, or three loaves and a twist, or three white loaves and a half-loaf of brown, and whether she shall take three quarts of milk or a pint more or less. Thus, whatever the terms on which alternatives are offered to us may be, we detect in conscious action at the margin of consideration the principles which are unconsciously at work in the whole distribution of our resources.”

Do you find anything to criticize in this?

(b) “When the supply (of a given commodity) is limited, the diminishing utility of each increment will be arrested at a point below which the consumer will prefer to abandon the use of an increment for something else. The margin here is a margin of indifference between an increment of one commodity and an increment of another commodity. Since these increments are not necessarily the same, the margin of indifference may be reached at a point where the tenth increment of one commodity balances the twentieth of another, where, in other words, the marginal utility of the first commodity is twice that of the second.”

Explain what you think is meant; and give your opinion on the conclusion stated in the last clause of the final sentence.

  1. “An English ruler who looks upon himself as the minister of the race he rules (say in India) is bound to take care that he impresses their energies in no work that is not worth the labor that is spent on it; or, to translate the sentiment into plainer language, that he engages in nothing that will not produce an income sufficient to defray the interest on its cost.”

Would Marshall question this principle? On what grounds, if at all? Would you?

 

Final Exam
Economics 11

Arrange your answers in the order of the questions.
Answer all the questions.

  1. “What about the ‘supply curve’ that usually figures as a determinant of price, coördinate with the demand curve? I say it boldly and baldly: there is no such thing. When we are speaking of a marketable commodity, what is usually called the supply curve is in reality the demand curve of those who possess the commodity; for it shows the exact place which every successive unit of the commodity holds in their relative scale of estimate.”

Is this criticism just if directed to (1) the temporary equilibrium of supply and demand, as analyzed by Marshall for a grain market; (2) the “price zone determined by marginal pairs,” as analyzed by Böhm-Bawerk; (3) the long period equilibrium of supply and demand, as analyzed by Marshall.

  1. “The rent of land is no unique fact, but simply the chief species of a large genus of economic phenomena; and the theory of rent is no isolated economic doctrine, but merely one of the chief applications of a particular corollary from the general theory of demand and supply.”

Explain this statement of Marshall’s; mention other species which he assigns to the large genus; and consider wherein, if at all, the general doctrine differs from that of Clark, and from that of Böhm-Bawerk.

  1. “As is true of good will and credit extensions generally, so with respect to the good will and credit strength of these greater business men: it affords a differential advantage and gives a differential gain. In the traffic of corporation finance this differential gain is thrown immediately into the form of capital and so added to the nominal capitalized wealth of the community. . . .This capitalization of the gains arising from a differential advantage results in a large ‘saving’ and increase of capital.”

Does this resemble in essentials Walker’s doctrine? If so, wherein? If not, why not?
In what sense, if in any, is it true that the differential gains lead to an increase of capital?

  1. “It may be conceded that if a certain class of people were marked out from their birth as having special gifts for some particular occupation, and for no other, so that they would be sure to seek out that occupation in any case, then the earnings which such men would get might be left out of account as exceptional, when we are considering the chances of success or failure for ordinary persons.”

Consider whether, given the premise, the conclusion here stated would follow; what the bearing of the reasoning is on Walker’s theory of business profit; what Marshall would say of premise and conclusion.

  1. In what sense, if in any, is a “productivity” theory of wages put forth by Walker? by Clark? by your instructor?
  2. “All capital goods — tools, machines, and the like — were explained [by the economists of the British School] as merely so much stored-up labor, or as the stored-up wages paid for it; the capitalist, as a laborer gone to seed; and thereby the product of capital as indirectly the product of the earlier wage-paid labor; interest being thus mere indirect wages. It was implied in this that the interest payments are for mere wear-out of the principal invested, and that the sum of all the interest payments upon a given investment can normally or regularly equal only the original capital sum invested in wages; and that sometime a given capital investment must cease its career of earning interest.”

Consider whether this was the doctrine of the British economists; whether it is the doctrine of Böhm-Bawerk; of your instructor; and give your own opinion.

  1. “In the main, the way in which the increase of savings can find escape from its difficulties is through the parallel advance in the arts, calling for more and more elaborate forms of capital. . . . Given continued improvements calling for more and more elaborate plant, —more of time-consuming and roundabout applications of labor, — than savings can heap up, and a return will be secured by the owner of capital.”

What are the ” difficulties ” here referred to? What would be said of this way of escape by Böhm-Bawerk? by your instructor? by Veblen?

________________________________

History and Literature of Economics to the year 1848

Course Description
Economics 14

[Economics] 14. History and Literature of Economics to the year 1848. Mon.,

Wed., and (at the pleasure of the instructor) Fri., at 11. Professor BULLOCK.

The purpose of this course is to trace the development of economic thought from classical antiquity to the middle of the nineteenth century. Emphasis is placed upon the relation of economics to philosophical and political theories, as well as to political and industrial conditions.

A considerable amount of reading of prominent writers will be assigned, and opportunity given for the preparation of theses. Much of the instruction is necessarily given by means of lectures.

 

Final Exam
Economics 14

  1. What significant analyses of economic structure were made by Aristotle, the Schoolmen, John Hales, Cantillon, and Smith?
  2. What do you consider the most significant analyses of economic functions made by Aristotle, the Schoolmen, Mun, Cantillon and the Physiocrats?
  3. Trace the development, in economic theory, of the idea of a beneficent natural order.
  4. What elements contributed to the economic system of Adam Smith, and what was Smith’s own contribution?
  5. Compare Ricardo’s economic theories with those of Smith.
  6. Trace the development of theories of money in the writings of Aristotle, the Schoolmen, the Mercantilists, and Ricardo.

________________________________

 

Topics in the Economic History of the Nineteenth Century

Course Description
Economics 24

[Economics] 24. Topics in the Economic History of the Nineteenth Century.

Two consecutive evening hours per week, to be arranged with the instructor. Professor GAY.

This course is designed to offer an opportunity for further study to graduate students who have taken or are taking Economics 2a and 2b. Reading will be assigned and reports presented for discussion on such topics as the spread of the Industrial Revolution to the Continent and the United States, the agrarian changes in England in the first half of the century, and in the second half-century the effects of American agricultural competition on the chief European countries, the history of transportation, with especial reference to problems of government ownership in Europe. Emphasis will be given to the comparative development of typical industries both in Europe and the United States, and changes in wholesale and retail organization.

Students who are taking at the same time this course and the lectures in Economics 2a and 2b may receive credit for one and a half courses.

 

Final Exam
Economics 24

  1. “Such has been the rage for Western immigration for the last twenty years that the soil of New England has, in the estimation of good judges, been greatly undervalued.” (From address before the Essex Agricultural Society, 1833.)

Is this statement true, and, if true, what were the chief causes?

  1. Outline the chief topics you would discuss in writing a monograph on agriculture in the United States during the period 1825 to 1845. Characterize the chief available sources of evidence.
  2. Describe briefly the canal systems of Massachusetts and New York. Compare the reasons for their construction and for their decline.
  3. Explain the Suffolk Banking System and discuss its effectiveness from 1830 to 1843.
  4. What statistical material would you use in studying the crisis of 1837-39? How does it compare in extent and value with that available for the crisis of 1907?

________________________________

 

Public Finance

Course Description
Economics 31

[Economics] 31. Public Finance. Mon., Wed., and (at the pleasure of the instructor) Fri., at 10. Professor BULLOCK.

The course is devoted to the examination of the financial institutions of the principal modern countries, in the light of both theory and history. One or more reports calling for independent investigation will ordinarily be required. Special emphasis will be placed upon questions of American finance. Ability to read French or German is presupposed.

 

Final Exam
Economics 31

  1. How far, in your opinion, does the general income tax conform to Smith’s canons of taxation?
  2. Compare local taxation in Great Britain with local taxation in either France or Germany.
  3. Discuss the incidence of taxes upon real estate.
  4. What, in your opinion, are the leading principles that should govern the distribution of taxation?
  5. What opinions concerning indirect taxation are held by the following writers: Smith, Bastable, and either Leroy-Beaulieu or Eheberg?
  6. Outline what you would consider a practicable plan for the reform of state and local taxation in the United States.
  7. Discuss the theory and practical operation of sinking funds.
  8. Describe the German system of product taxes. What does Eheberg think of the system?
  9. What is Leroy-Beaulieu’s opinion of the changes effected in French taxation during the last twenty years, and what changes does he advocate?

Answer the questions in order. Omit either the eighth or ninth question.

 

 

Sources:

Harvard University Examinations. Papers Set for Final Examinations in History, History of Science, Government, Economics, Philosophy, Psychology, Social Ethics, Education, Fine Arts, Music in Harvard College (June, 1914), pp. 38-54.

Mid-year exams for Economics A and Economics 11 from Harvard University Archives: Examination papers in economics, 1882-1935, Scrapbook of Prof. F. W. Taussig. (HUC 7882).

Harvard University. Division of History, Government, and Economics, 1913-14. Official Register of Harvard University, Vol. X, No. 1, Part X (May 19, 1913).

 

 

Categories
Exam Questions Harvard

Harvard. Final Examination Questions. Economics Courses, 1912-13

 

 

For the academic years 1912-13 through 1915-16 there are complete (or at least nearly complete) sets of examinations for many departments, including economics available at hathitrust.org. In this posting we have final examinations for all economics courses but three for the 1912-13 academic year. Since courses are only identified in these collections by number, I have provided the course titles, instructors’ names and course registration figures available in the annual Harvard Presidential Report for that academic year.

The three courses for which no final examination questions (perhaps the grade was not even determined by examination) were:

Economics 13. Statistics. Theory, method, and practice. Professor Ripley. (6 Graduates, 1 Senior, 3 Radcliffe: Total 10)

Economics 24. Topics in the Economic History of the Nineteenth Century. Professor Gay. (4 Graduates, 1 Senior: Total 5)

Economics 33 1hf. Tariff Problems in the United States. Professor Taussig. (5 Graduates, 3 Seniors: Total 8)

FINAL EXAMINATIONS
1912-13

Economics 1. Principles of Economics
Economics 2a lhf. European Industry and Commerce in the Nineteenth Century
Economics 2b 2hf. Economic and Financial History of the United States
Economics 3. Money, Banking, and Commercial Crises
Economics 4a 1hf. Economics of Transportation
Economics 4b 2hf. Economics of Corporations
Economics 5. Public Finance, including the Theory and Methods of Taxation
Economics 6a lhf. Trade-Unionism and Allied Problems
Economics 6b 2hf. The Labor Movement in Europe
Economics 7. Theories of Distribution and Distributive Justice
Economics 8. Principles of Sociology
Economics 9. Principles of Accounting.
Economics 11. Economic Theory
Economics 12 lhf. Scope and Methods of Economic Investigation
Economics 14. History and Literature of Economics to the year 1848
Economics 16. The History of Modern Socialism
Economics 23. Economic History of Europe to the Middle of the Eighteenth Century
Economics 31. Public Finance
Economics 32 2hf. Economics of Agriculture, with special reference to American conditions

 

________________________________________

Economics 1. Principles of Economics.

Professor Taussig and Dr. E. E. Day, assisted by Messrs. Heilman, Jones, Burbank, Crosgrave, and Eldred.
1 Graduate, 21 Seniors, 93 Juniors, 307 Sophomores, 21 Freshmen, 38 Other. Total 481.

 

[p. 38-39]

ECONOMICS 1

  1. To what extent and in what manner do the following contribute to the formation of capital: (a) a government loan; (b) the stock exchange; (c) commercial banks; (d) the corporate form of business organization?
  1. Explain “margin of cultivation.” Distinguish between the intensive margin of cultivation and the extensive margin of cultivation. What is the relation between (a) the margin of cultivation in agriculture and the price of a bushel of wheat; (b) the margin in gold mining and the value of an ounce of gold?
  1. Assume that a monopoly produces a commodity under conditions of constant cost. What determines the extent to which the monopoly price will be above the price which competition, if existent, would establish? Illustrate by diagram.
  1. The rentals from a New York office building amount to $50,000 a year. The building is worth $200,000. To provide for insurance, depreciation and such fixed items, $10,000 is expended annually. The current rate of interest upon investments of equal security is 5%. What is the value of the land?
  1. What is “dumping “? What induces it? To what extent is it dependent upon (a) monopoly conditions; (b) tariff barriers?
  1. Explain briefly why (a) the rates of wages are generally higher in the United States than in Germany; (b) higher for plumbers than for unskilled laborers; (c) for domestic servants than for women employed in shops and factories. Suppose a socialist community apportioning wages on the basis of equality of sacrifice: would these differences persist?
  1. How are the wages and the number employed within a particular industry affected, immediately and ultimately, by the invention of labor-saving devices in that industry?
  1. Explain: (a) railroad rebates; (b) over-capitalization; (c) public service industries; (d) “reasonable restraint of trade”; (e) stoppage at the source.

 

________________________________________

 

Economics 2a lhf. European Industry and Commerce in the Nineteenth Century.
Professor Gay, assisted by Dr. M. T. Copeland.
16 Graduates, 14 Seniors, 44 Juniors, 17 Sophomores, 3 Freshmen, 5 Other. Total 99.

 

[p. 39-40]

ECONOMICS 2a1

  1. “The effect of Peel’s measures of 1842-1845 was to demonstrate how much the trade and industry of the country might be encouraged by the readjustment of fiscal burdens.” Explain.
    Is a similar readjustment needed in England at the present time? Why or why not?
  1. (a) How was the capital for the construction of railroads prior to 1870 obtained in the different European countries? Why?
    (b) Why was the railroad policy of Prussia modified after 1870? With what results?
  1. Compare the organization of the wool manufacturing industries in England, Germany, and France at the present time, explaining to what the differences are due. How far are these differences typical?
  1. Compare the English and Belgian methods of relieving the recent agricultural depression.
  1. In which industries have Kartells been formed in Germany? Why? Compare with the movement for combination in England.
  1. Explain the statement in regard to English agriculture that “after the middle of the eighteenth century the two revolutions, the industrial and the agricultural, which are indeed only manifestations of the same scientific and commercial spirit, go hand in hand and supplement one another.” Does this statement apply also to Germany? Why or why not?
  1. Discuss briefly —

(a) English ” Friendly Societies.”
(b) Pitt’s Sinking Fund.
(c) Zollverein.
(d) French shipping subsidies.
(e) Charter and line traffic.

 

________________________________________

 

Economics 2b 2hf. Economic and Financial History of the United States.
Professor Gay, assisted by Dr. M. T. Copeland.
18 Graduates, 22 Seniors, 50 Juniors, 27 Sophomores, 1 Freshman, 6 Other. Total 124.

 

 

[p. 40-1]

ECONOMICS 2b

  1. “The most important feature of life in a newly settled community is its commercial connection with the rest of the world.” Why? How is this illustrated (a) by the history of the American colonies and (b) by the history of the West?
  1. What were the causes for the decline of the American merchant marine? What attempts have been made to assist its recovery? With what results?
  1. Compare in its main features the economic history of the decade 1830-40 with that of the decade 1880-90.
  1. Compare the conditions which stimulated industrial combinations in the ’90’s with those which resulted in railroad combinations in the ’70’s.
  1. Within the last twelve months the New England Cotton Yarn Company, the U. S. Finishing Company, and the International Cotton Mills Corporation have each undergone reorganization. What was the earlier history of these companies and how far did that history foreshadow the necessity for such reorganizations?
  1. If you were to establish a mill for manufacturing silk goods at the present time, how would the conditions which you would meet in that industry differ from those which confronted a silk manufacturer forty or forty-five years ago? Why have these changes taken place? How far are they typical of the general industrial development of the United States during this period?
  1. (a) Comment on the following statement, which was made in a speech in Congress in 1846. “It is a protective tariff which gives to American industry the only effectual guaranty that it will not be brought down to a level with the degraded labor of Europe. It furnishes the only security that our standard of wages is not to be measured by the cost of production in those countries where the life of the laborer is but an incessant struggle for bread.”
    (b) Judging from the history of the years 1893-1900, as well as from present conditions, is the present year more or less opportune than 1909 for a downward revision of the tariff? Why?

 

________________________________________

 

Economics 3. Money, Banking, and Commercial Crises.
Dr. E. E. Day, assisted by Messrs. Ise and F. E. Richter.
3 Graduates, 31 Seniors, 67 Juniors, 16 Sophomores, 1 Freshman, 1 Other. Total 119.

 

[p. 41-42]

ECONOMICS 3

  1. What factors favored the monetary rehabilitation of silver in the United States during the 70’s? Which of these factors are still operative? Explain the disappearance of the others.
  1. What banking abuses were most common in the United States early in the nineteenth century? When and how, if at all, have these since been eliminated?
  1. What is the relation between the Bank of England rate and the London market rate of discount when (a) funds are abundant; (b) funds are relatively scarce? In what ways does varying the Bank rate accomplish the protection of the English banking reserves?
  1. What is meant by a free gold market? Are the following such: London; Paris; Berlin; New York? In each case, why or why not?
  1. How will the rate of sterling exchange in New York be affected by: (a) a crop failure in the United States; (b) hoarding of specie in Europe; (c) a slump on the New York Stock Exchange; (d) a banking panic in this country?
  1. “The call-loan market . . . furnishes to the banks of the country under the present organization of banking, their only means of mobilizing their reserves, of liquifying their assets, and of securing flexibility in their lending power.” Explain and criticize. How, if at all, should this feature of our system be changed?
  1. In the equation of exchange given by Professor Fisher, what is the relation of M, M’, V, and T (a) during a period of rising prices; (b) during a period of settled prices?
  1. Describe the crisis of 1873 with reference to: (a) its general antecedents; (b) its more important causes in the United States; (c) its final outbreak in this country; (d) the territorial extent of the reaction; (e) the severity and duration of the subsequent depression.

 

________________________________________

 

Economics 4a 1hf. Economics of Transportation.
Professor Ripley, assisted by Mr. Crosgrave.
6 Graduates, 2 G.B., 36 Seniors, 85 Juniors, 24 Sophomores, 3 Other. Total 156.

 

[p. 42-3]

ECONOMICS 4a

  1. Compare the lease with stock ownership as a means of combination, stating the advantages and disadvantages of each.
  1. Show how recent interpretation of the Federal law may conceivably affect the status of railway traffic agreements.
  1. State very briefly the point raised in the following cases: —

(a) Portland Gateway.
(b) Illinois Central car supply.
(c) Alabama Midland (Troy).
(d) Orange Routing.

  1. Have any of the above points been since corrected by legislation; if so in what manner?
  1. Give reasons for the following differences in net capitalization per mile of line:

Union Pacific $65,000         Reading $170,000
Pennsylvania    86,000        Erie            170,000

  1. What particular circumstance materially affects the success of Government ownership and operation:

(a) In Germany?
(b) In Italy?
(c) In Switzerland?

  1. What is the present attitude of the Federal courts toward the proper basis to be used in the determination of reasonable rates?
  1. How effective practically has been the ” Commodity Clause” of the law of 1906?
  1. To whom properly belongs the surplus earnings of a railroad over and above a rate of return requisite to provide an adequate supply of new capital for future development? State your own view, but set forth your reasons fully.

 

________________________________________

 

 

Economics 4b 2hf. Economics of Corporations.
Professor Ripley, assisted by Mr. Crosgrave.
6 Graduates, 20 Seniors, 86 Juniors, 15 Sophomores, 3 Other. Total 130.

 

 

[p. 43-4]

ECONOMICS 4b

  1. Why was the dissolution of the “Tobacco Trust” more difficult than that of the Standard Oil Company? Explain fully.
  1. Indicate certain differences in the eye of the law between monopolization and restraint of trade.
  1. Herewith are two balance sheets of companies A and B respectively. Comment upon them, contrasting one with the other. Which apparently denotes the greater financial stability?
Co. A
Assets Liabilities
Plant $3,500,000 Preferred Stock $5,000,000
Merchandise 1,800,000 Common Stock 15,000,000
Bills Receivable 700,000 Accounts Payable 600,000
Cash 1,400,000
Good-will and Patents 13,200,000
$20,600,000 $20,600,000
 

Co. B

Assets Liabilities
Factories $15,000,000 Capital Stock $65,000,000
Securities owned 18,000,000 Debentures 15,000,000
Merchandise 20,000,000 Surplus 3,000,001
Accounts Receivable 30,000,000
Franchises and Good-will 1
$83,000,001 $83,000,001

 

  1. Name, with briefest possible description in each case, and in order of seriousness, at least five distinct forms of unfair competition in trade.
  1. Do all the foregoing forms of unfair competition affect thgeneral public as well as direct competitors? Does this factor apparently influence the attitude of the courts?
  1. What was the essence of the U. S. Steel Bond Conversion plan? What became of it?
  1. Contrast administrative and judicial forms of controlling monopoly, pointing out the merits of each.
  1. Outline the plan of reorganization of the National Cordage Company. Was it typical of industrial reorganizations in general?
  1. What are the three leading objections to the so-called “holding company “?
  1. Outline what most appeals to you as a feasible plan for dealing with the existing trust problem. State concisely in definite propositions covering all points at issue.

 

________________________________________

 

Economics 5. Public Finance, including the Theory and Methods of Taxation.
Professor Bullock.
6 Seniors, 14 Juniors, 4 Sophomores, 1 Other. Total 25.

 

[p. 44]

ECONOMICS 5

  1. Explain and discuss critically the methods employed in the taxation of land in Germany, France, Great Britain, Australia, and the United States.
  1. Compare the general property tax with the general income tax, considering both the theory and the practical operation of these taxes.
  1. Compare the French, Prussian, and British systems of direct taxation.
  1. Compare the British system of indirect taxation with those of France and the United States.
  1. Discuss the taxation of mortgages in the United States.
  1. What changes in the taxation of personal property have recently occurred in the United States?
  1. Compare the British, Prussian, and Italian income taxes. .
  1. Outline what you consider a satisfactory theory of the just apportionment of public charges.

 

________________________________________

 

Economics 6a lhf. Trade-Unionism and Allied Problems.
Professor Ripley, assisted by Mr. Crosgrave.
3 Graduates, 44 Seniors, 19 Juniors, 4 Sophomores, 2 Other. Total 72.

 

[p. 45]

ECONOMICS 6a

Answer the first five briefly

  1. What is sabotage?
  1. What is the ” extended ” closed shop?
  1. What is the principal practical difficulty in the “general strike”?
  1. Is it met by the adoption of any positive policy in France by the “syndicates”?
  1. In the syndicalist programme what is to be the unit in the reorganized state?
  1. Contrast collective bargaining under sanction of the law with its adoption by private arrangement; (a) from the point of view of advantage to the employer; (b) from that of the workman.
  1. What are the four main features of the New Zealand legislation. (Each in a sentence.)
  1. What is the principal demonstrated weakness in the above legislation?
  1. What are three disabilities of the individual workmen in negotiating a wage contract?
  1. Wages for women in domestic service and in manufactures seem out of line with one another. What main difference helps to explain this?
  1. What is the present condition of affairs respecting the closed shop in the United States? Outline the course of events for two decades.
  1. How does the law of conspiracy enter into the decision by courts in labor disputes? How has Great Britain settled it?

 

________________________________________

 

Economics 6b 2hf. The Labor Movement in Europe.
Asst. Professor Rappard.
5 Graduates, 12 Seniors, 9 Juniors, 1 Sophomore, 3 Other. Total 30.

 

[p. 45-6]

ECONOMICS 6b

Arrange answers in order of questions. Students who wrote theses will omit the first three questions.

  1. Enumerate five of the effects which Engels says the Industrial Revolution had on the manufacturing population of England. What were Engel’s chief sources of information?
  1. How does Sombart distinguish between (a) Rational Socialism (Utopian Socialism and Anarchism) and (b) Historical Socialism?
  1. What effect, according to Marx, does machinery have

(a) Upon real wages?
(b) Upon nominal wages?
(c) Upon “relative surplus-value”?
(d) Upon “absolute surplus-value”?

  1. Why is it customary to mention the English enclosure movement in dealing with the history of labor in Europe in the 19th century?
  1. What were the historical relations between the doctrines of Godwin, Malthus and Darwin?
  1. What was Chartism? Saint-Simonism? Which was more radical? More socialistic? Give reasons.
  1. Write a biography of Marx (300 to 500 words).
  1. Compare the views of Marx and Vaudervelde on ” Capitalist Concentration.”
  1. Give chapter headings of a thesis on “The Socialist Movement in Germany, 1860-1890” in six or more chapters.
  1. Distinguish between (a) Socialism (b) Anarchism (c) Syndicalism.
  1. “From each according to his abilities, to each according to his needs … To every laborer the entire product of his labor … At first sight, these two formulas are absolutely contradictory. We believe, however, that it is possible and necessary to reconcile them and to complete each by the other.” — Vaudewelde.
    How does the author do this? What practical suggestions does he make for arranging distribution in the socialist state?
  1. What difficulties does Skelton think a socialist state would encounter

(a) In administering the government?
(b) In determining what commodities should be produced?
(c) In distributing wealth?

 

________________________________________

 

Economics 7. Theories of Distribution and Distributive Justice.
Professor Carver.
3 Graduates, 7 Seniors, 13 Juniors, 1 Sophomore, 1 Other. Total 25.

 

[p. 47]

ECONOMICS 7

  1. Explain and illustrate the principle of marginal utility and its relation to the value of consumers’ goods.
  1. Explain and illustrate the law of variable proportions and its relation to the value of the factors of production.
  1. Discuss the various criteria of justice in the distribution of wealth.
  1. Explain and illustrate exactly what you understand by self-interest.
  1. How would the single tax probably affect the demand for labor? Would its effect probably be stronger on unskilled than on skilled labor? On skilled labor than on business talent?
  1. How do mechanical inventions affect the demand for capital and for different grades of labor?
  1. Describe one communistic society, giving some account of its origin, the causes of its success if it succeeded and of its failure if it failed.
  1. Outline a program for raising the wages of all the lower grades of labor.

 

________________________________________

 

Economics 8. Principles of Sociology.
Professor Carver.
10 Graduates, 41 Seniors, 74 Juniors, 18 Sophomores, 4 Other. Total 147.

 

[p. 47-8]

ECONOMICS 8

  1. What, in your opinion, is the ultimate test of progress? Give your reasons.
  1. Compare the views of Buckle and Peschel as to the influence of geographical surroundings on religion.
  1. What is the relation between the institution of the family and the institution of property?
  1. What place has the genius in social progress? Give your own opinion and state the views on this question of authors whom you have read.
  1. Outline the leading forms of waste labor and of waste land, giving briefly the reasons why each form of waste exists at the present time.
  1. Compare the views of Mill and Ross as to the limits of social control.
  1. What, according to Ross, is the relation of resentment to social order?
  1. What are the reasons for the existence of the ballot? How far would these reasons justify the extension of the ballot?

 

________________________________________

 

Economics 9. Principles of Accounting.
Asst. Professor Cole, assisted by Mr. Eliot Jones.
7 Graduates, 8 Graduates of Applied Sciences, 62 Graduates of Business School, 147 Seniors, 50 Juniors, 2 Sophomores. Total 276.

 

[p. 48-50]

ECONOMICS 9

Save one hour for the last question. It will count as one-third of the paper.

  1. Show by journal entries what should be debited and what credited for the following transactions:

(a) Granting a discount to a customer, for early payment of a bill, so that, though the amount of the bill was $100, he pays but $95.
(b) Paying a lawyer $50 for trying to collect a bill that proves uncollectible, and writing off the debt ($250) as bad.
(c) Collecting $75 as full payment, including interest to the amount of $17, for a debt previously written off as bad.
(d) Giving a friend whose credit at banks is not very good, because he is a new-comer in town, and for whom, therefore, you do not wish to endorse notes, your own note for $1000, with the understanding that he will discount it at a bank, and taking in exchange your friend’s note (for the same amount and time) which you intend to keep until maturity.
(e) Discounting at a bank your friend’s note mentioned in (d), because you find his credit has improved in the public mind and you need the money. [Discount $7.]
(f) Returning to the manufacturers, as unsatisfactory, goods billed at $500 and bought to be sold at $650.
(g) Delivering goods from the store as part payment of clerks’ wages, and allowing 5% discount to clerks. [Retail price $50, clerks’ price $47.50.]
(h) Issuing a stock dividend of $50,000.
(i) Selling a new $2,000,000 issue of stock for $2,100,000. [Corporation’s books.]

  1. A bond table gives the value of $10,000 of bonds for January 1 as $10,366.27, and for July 1 as $10,323.60. On the latter day you collect $250 interest. What entry shall you make for the interest?
    Assuming that the valuation of the bonds was determined on a 4% basis, how could you prove the correctness of the July 1 valuation if you knew that the valuation for January 1 was correct?
  1. Define and discuss the purpose of the following: —

(a) a machine rate,
(b) a life-insurance reserve,
(c) a national-bank redemption fund,
(d) a stores ledger,
(e) a machine ledger,
(f) a controlling account.

  1. Would expense burden enter into a plan of cost accounting for (a) a department store, (b) a hospital, (c) a college, (d) a gas company? Explain briefly how, or why not, in each case.

Remember in solving problems that time and confusion are often saved by the use of journal entries as guides in determining which accounts are affected.

  1. The balance sheet a year ago was as follows: —
Plant $125,000 Capital Stock $140,000
Accounts Receivable 33,000 Bills Payable 10,000
Merchandise 19,000 Accounts Payable 24,000
Cash 5,000 Surplus 8,000
$182,000 $182,000

An abbreviated tentative income sheet for the year just closing gave the following figures: —

Wages $85,000 Other Expenses $71,000
Materials 54,000 Gross Income 240,000

No items relating to the care of property were included in the “other expenses,” and they are now to be provided for. Such items are found on the debit side of the trial balance as follows:—

Depreciation $5,000 Replacement $4,000
Repairs 8,000 Additions 12,000

Supposing the only changes in the balance sheet are those caused by the items shown above (profit or loss and care of property) and that cash absorbs the net effect of changes not otherwise indicated, show the income sheet and the balance sheet for the new year.

  1. Prepare such a tabular statement or statements as an accountant should give to his employers or clients for a business yielding the following figures on three trial balances (of ledger balances) taken at the times indicated.
Trial balance at the opening of business, Jan. 1, 1912 Trial balance, Dec. 31, 1912, before the books are closed Trial balance at the opening of business, Jan. 1, 1913
Dr. Cr. Dr. Cr. Dr. Cr.
Capital Stock $200,000 $200,000 $200,000
Bills Payable 30,000 40,000 40,000
Accounts Payable 35,000 37,500 37,500
Surplus 7,000 7,000 9,000
Dividends declared 10,000 10,000
Real Estate and Plant $135,000 $137,500 $137,000
Accounts Receivable 88,200 80,200 80,200
Goods in process 17,000 17,000 20,000
Finished Goods 25,000 25,000 23,000
Raw Materials Inventory 15,000 15,000 35,000
Raw Materials 57,000
Wages 7,000 52,000 2,000
Taxes 200 2,300 200
Insurance 1,000 2,200 1,000
General Expenses 7,500
Sales 113,200
Cash 8,000 2,000 2,000
$289,200 $289,200 $397,700 $397,700 $298,700 $298,700

If you give more than one statement, prepare one at a time, and leave the reconciliation between statements until all are complete.

 

________________________________________

 

Economics 11. Economic Theory.
Professor Taussig.
20 Graduates, 1 Graduate at Business School, 4 Seniors, 5 Juniors, 1 Other. Total 31.

 

[p. 50-3]

ECONOMICS 11

[Arrange your answers strictly in the order of the questions]

  1. Explain the connection between

(a) the rent of mines;
(b) Carey’s doctrine that the total rent received by land-owners is less than interest on the total investment for improving land;
(c) the earnings of barristers or opera-singers;
(d) the earnings of ” successful ” business men.

  1. “Men are not equal. . . . Those capable of organizing and leading industrial enterprise are in a minority, and are indeed few; hence they can put a price on their services which would be impossible if there were many. Their services are not worth more on this account, but they can get more for them. Because the community needs their services, and cannot perhaps get along without them, they can, if they like, put ” famine prices ” on the commodity (organizing and directing talent) which they have to sell; while, on the other hand, those who have only labor or physical skill, though they are just as necessary, are many, and hence can about as readily be taken advantage of as the others can take advantage.” What have you to say? Can the ” famine prices ” be justified?
  1. (a) “There are, in fact, few no-rent men in actual employment; and the reason for this is clear, since work involves a sacrifice, and it does not pay to incur the sacrifice unless the earnings be a positive quantity. In those times and places in which child labor has been employed, with little regard for the welfare of the victims, labor that was not at the no-rent point, but very near it, has been pressed into service. But, where the sacrifice entailed by labor is, in some way, neutralized by a benefit that work confers, labor which creates literally nothing may sometimes be employed. Lunatics or prisoners may be kept at work, in order that they may secure fresh air and exercise, even though the amount of capital that they use, if it were withdrawn from their hands and turned into marginal capital, would produce as much as it does when it is used by them. In such a case the product imputable to their labor is nil.
    The existence of any no-rent labor enables us to make the rent formula general and to apply it to every concrete agent of production.”
    (b) “The productivity of any capital, whether human or external, will differ with the capital. Men differ in quality, i. e., in productive power, as truly as lands or other instruments differ. Some men have a high degree of earning power and some have not.
    Some men can work twice as fast as others. Some men can do higher grades of work than others. The result is that we find men classified as common manual laborers, skilled manual laborers, common mental workers, superintending workers, and enterprisers.
    Just as we can measure the rent of any land by the difference in productivity between that and the low-rent, or no-rent, land, in exactly the same way we can measure the difference in productivity between men. There is no grade of workmen called the “no-wages men,” but there would be such a grade if it were customary for their employer to pay for their cost of support (as the employer of land pays for its cost), so that only the excess above this cost were to be called wages.”

Compare the two trains of reasoning; give your opinion; and state by what authors the passages were written.

  1. “If the proprietor of superior land were to say, ‘I will take no rent for it,’ this would not make wheat cheaper. The supply would not be changed; for the same quantity would be raised, the marginal amount raised on the no-rent land would be needed and would be bought at the former price, and all other parts of the supply would command the same rate. … It is a striking fact — but one hitherto much neglected — that similar conclusions apply to the product of every other agent ” [capital and labor]. Do similar conclusions apply? Who do you think is the author of this passage?
  1. What three grounds explain, according to Böhm-Bawerk, the preference for present goods over future? Which of them does he conclude to be the most important? State Fisher’s criticism; and give your own opinion on the controverted question.
  1. “In the present condition of industry, most sales are made by men who are producers and merchants by profession. . . .For them, the subjective use value of their own wares is, for the most part, very nearly nil. … In sales by them the limiting effect which, according to our theoretical formula, would be exerted by the valuation of the last seller, practically does not come into play.” — Böhm-Bawerk.
    What is the ” theoretical formula “? and what is the importance of the qualification here stated?
  1. In what sense are the terms “demand” and “increase of demand ” used in the following passages:

(a) “The democratization of society and the aping of the ways of the well-to-do by the lower classes have greatly increased the demand for silk fabrics.”
(b) “The lower price of sugar after 1890, when sugar was admitted free of duty, at once caused an increase of demand.”
(c) “The cheapening of a commodity may mean an increase of demand such that the total sum spent on it will be as great as before, even greater than before.”

  1. Explain the essentials of Veblen’s theory of crises, and state wherein you think it most tenable, wherein least so.

 

________________________________________

 

Economics 12 lhf. Scope and Methods of Economic Investigation.
Professor Carver.
2 Graduates, 1 Radcliffe. Total 3.

 

[p. 53]

ECONOMICS 12

  1. Explain verbally, and show by means of an outline, the relation between private and public economics and the main subdivisions of each.
  2. Into what main departments would you subdivide the subject of economics if you were going to write a general text book for college classes. Give your reasons.
  1. What are the characteristic methods of reasoning, methods of collecting information, and methods of exposition in economics? Mention examples, or give illustrations of each. What are the special advantages of each? To what class of problems is each especially adapted?
  1. Comment upon the following: —

“The economist may thus be considered at the outset of his researches as already in possession of those ultimate principles governing the phenomena which form the subject of his study, the discovery of which in the case of physical investigation constitutes for the inquirer his most arduous task; but, on the other hand, he is excluded from the use of experiment.” (Cairnes, pp. 89-90.)

  1. What, according to Warner, are the characteristic methods of determining the causes of poverty? What are the merits and defects of each method? Give illustrations.
  1. Comment upon the statement that “political economy depends more upon reasoning than on observation.” Is this the same as saying that the greatest present need is for sound reasoners rather than for close observers? Would either statement apply to all possible conditions and to all classes of problems?
  1. Discuss Clark’s reasons for describing capital as a sum of money.

 

________________________________________

 

Economics 14. History and Literature of Economics to the year 1848.
Professor Bullock.
7 Graduates. Total 7.

 

[p. 54]

ECONOMICS 14

  1. What significant analyses of economic structure and functions were made by the mercantilists?
  1. Discuss the development of economic opinions as reflected in the writings of Hales, Bodin, Montchrétien, Mun, Petty, Boisguilbert, Cantillon, Vanderlint, and Hume.
  1. Explain the structure and purpose of the “Wealth of Nations,” and give a brief analysis of the doctrines of the first two books.
  1. Discuss the treatment of the subject of population by Aristotle, the Schoolmen, Cantillon, Smith, and Malthus.
  1. At what points did the economic theories of Ricardo differ from those of Adam Smith?

 

________________________________________

 

Economics 16. The History of Modern Socialism.
Asst. Professor Rappard.
4 Graduates Total 4.

 

[p. 54-5]

ECONOMICS 16

  1. Fill out the blanks in the following table according to the Marxian phraseology and theory.
Con-stant capital Vari-able capital Rate of surplus value Capital con-sumed Indi-vidual rate of profit Value of commo-dities pro-duced Cost price of commodities prod-uced Average rate of profit Price of com-modities Deviation of price from value
90 10 50% 20
80 20 50% 10
70 30 50%
  1. “The theory of value which Marx presents is a variation of the familiar labor-value doctrine.” Discuss.
  1. State the Marxian theory of rent.
  1. What is meant by the Bernstein-Kautsky controversy? State three of the principal points involved, with the arguments advanced on both sides.
  1. What, according to Skelton, are the distinctive features of Utopianism? How does Shelton classify the Utopian doctrines?
    What, according to Skelton, are the two “quite distinct interpretations” of which the Marxian materialist conception of history is susceptible?
  1. “In spite of himself, Marx was the last of the classical economists.” How does Shelton justify this assertion?
  1. “Had the third volume of ‘Capital’ appeared at the same time as the first, little would have been heard about ‘exploitation’ from socialist platforms.” Why not, according to Skelton?

 

________________________________________

 

Economics 23. Economic History of Europe to the Middle of the Eighteenth Century.
Dr. Gray.
4 Graduates, 1 Radcliffe. Total 5.

 

[p. 55]

ECONOMICS 23

  1. Discuss the origin and early expansion of capital in Italy, the Low Countries, Germany and England. (One hour.)
  1. Compare the development of copyhold in England with that of Meierrecht in Germany. In what way were agrarian conditions in southwestern Germany different from conditions in the north-west at the beginning of the sixteenth century.
  1. Trace the growth of the mercantile system in England. Has Cunningham’s treatment any bias? Explain.
  1. Describe fully four of the following documents: —

Notularium Johannis Scribae.
An English Pipe Roll.
Royal licenses to export English wool in 1273.
De institutis Lundonie.
Chrysobullium Alexii I.

 

________________________________________

 

Economics 31. Public Finance.
Professor Bullock.
6 Graduates, 1 Junior. Total 7.

 

[p. 55-6]

ECONOMICS 31

  1. How far does the present British system of taxation conform to the maxims of Adam Smith?
  1. How far does the present French system of taxation conform to Smith’s maxims?
  1. How far does the present Prussian system of taxation conform to Smith’s maxims?
  1. How far would the single tax on land values conform to Smith’s maxims?
  1. Compare the general property tax in Switzerland with the same tax in the United States.
  1. What changes in the general property tax have occurred in the United States in recent years?
  1. Discuss fully the opinions of Leroy-Beaulieu or Eheberg concerning the income tax.
  1. Discuss fully the opinions of Leroy-Beaulieu or Eheberg concerning the inheritance tax.

 

________________________________________

Economics 32 2hf. Economics of Agriculture, with special reference to American conditions.
Professor Carver.
8 Graduates, 2 Seniors, 1 Junior. Total 11.

 

[p. 56]

ECONOMICS 32

  1. What are some of the larger characteristics which distinguish rural from urban life?
  1. Where would you draw the line between large scale and medium scale, and between medium scale and small scale farming, and what are the principal advantages and disadvantages of each?
  1. Exactly what is the distinction between intensive and extensive farming, and what are the advantages and disadvantages of each?
  1. To what system of culture does the horse as a draft animal belong, and what are some of the characteristics of that system?
  1. Where do we find the larger percentage of tenancy in this country, where land is highly productive or where its productivity is low? How would you explain the situation?
  1. Give your ideas as to the function of the middle-man, and to what extent and how that function may be performed by the farmers themselves.
  1. What are the advantages of diversified farming as compared with specialized farming?
  1. Give your ideas as to how country life may be made more attractive to men and women of education and culture.

________________________________________

Sources:

Harvard University Examinations. Papers set for final examinations in history, history of science, government, economics, philosophy, social ethics, education, fine arts, music in Harvard College. June, 1913. Cambridge, MA.

Harvard University. Reports of the President and the Treasurer of Harvard College, 1912-13, pp. 57-58.